Sie sind auf Seite 1von 15

Solutions to Problems in Peskin and Schroeder, An Introduction To Quantum Field Theory

Homer Reid March 13, 2003

Chapter 3
Problem 3.1
Part a. The commutation relations are [J , J ] = i g J g J g J + g J . Then we have [K i , K j ] = [J 0i , J 0j ] = i g i0 J 0j g 00 J ij g ij J 00 + g 0j J i0 We can replace g 00 = 1, J 00 = 0, and g i0 = g j 0 = 0 since we know i and j are space indices. Then [K i , K j ] = iJ ij = Next, [Li , K n ] = 1 2
ijk ijk

Lk .

[J jk , J 0n ] tensor to write

This is really just a fancy way of using the properties of the = 1 [J pq , J 0n ] [J qp , J 0n ] 2

Homer Reids Solutions to Peskin and Schroeder Problems: Chapter 3

where p = 1 + ((i + 1) mod 3) and q = 1 + ((i + 2) mod 3). In the second commutator we can switch q and p and simultaneously ip the sign since J is antisymmetric to obtain = [J pq , J 0n ] = i g q0 J pn g p0 J qn g qn J p0 + g pn J q0 = i g pn J q0 g qn J p0 . But only one of g pn and g qn can be nonzero, and whichever one is nonzero has the value 1 since p and q are space indices. So we get [Li , K n ] = iJ i+2,0 , +iJ i+1,0 , (n = i + 1) (n = i + 2)

(where all addition is to be carried out modulo 3). = i =i


ink

J k0

=i

ink ink

J 0k K k.

Finally, the last commutator is calculated through a calculation exactly analogous to the one we just went through, and actually the answer is given to us in the problem, namely, [Li , Lj ] = i Next, we have [J+ , J ] = 1 (L + iK), (L iK) 4 1 = [L, L] + i [K, L] i [L, K] + [K, K] 4 = 2i [K, L] = 2i [K 1 , L1 ] i + [K 2 , L2 ] j + [K 3 , L3 ] k =0 since [K i , Lj ] = 0 only for i = j as we saw above. Next,
j i J+ , J+ = ijk

Lk .

1 (Li + iK i ), (Lj + iK j ) 4 1 [Li , Lj ] + i[K i , Lj ] + i[Li , K j ] [K i , K j ] = 4 1 i ijk Lk + i ijk + i ijk K k + i ijk Lk = 4 1 j = ijk J+ 2

Homer Reids Solutions to Peskin and Schroeder Problems: Chapter 3

so the J+ operators satisfy the angular momentum commutation relations among themselves. The demonstration that the same is true for the J operators is just too similar to what we just did to warrant repeating. Part b.
1 , 0) representation of the Lorentz group we want to take For the ( 2

J+ =

1 1 (L + iK) = , 2 2

J =

1 (L iK) = 0. 2

i These are solved by taking L = 1 2 , K = 2 . Then the transformation law for 1 the ( 2 , 0) representation is

( 1 2 ,0)

= =

1 ei J ( 2 ,0) 1 ei(L+K) ( 2 ,0)

. e 2 2 ( 1 2 ,0)

1 For the (0, 2 ) representations we want to take

J+ =

1 (L + iK) = 0, 2

J =

1 1 (L iK) = 2 2

i 1 , K = 2 . Then for this object the transwhich is obtained by taking L = 2 formation law is i 1 (0, 1 e 2 + 2 (0, 1 . 2) 2)

Comparing with Peskin and Schroeder equation 3.37, this tells us to identify ( 1 = L , 2 ,0) Part c.
1 (0, 2 ) = R .

Parametrizing the matrix as suggested, we have


1 1 ( 2 ,2) =

V0+V3 V 1 + iV 2

= V

V 1 iV 2 V0 V3

where 0 = , i = the ith Pauli matrix. Then applying the transformation described, we obtain
e 2 (i+ )
1

2 (i ) 1 1 ( 2 ,2) e

1 1 1 (i + ) V 1 (i ) 2 2 1 1 = V (i + ) V (i ) V + O(2 ) 2 2 1 1 V V i {, } V i [, ] 2 2

Homer Reids Solutions to Peskin and Schroeder Problems: Chapter 3

But for the matrices we have the following properties: 0 , 0 = 2 0 i , 0 = 2 i i , j = 2 ij 0 , 0 = 0 i , 0 = 0 i , j = 2ieijk k so the above equation becomes e 2 (i+)
1

2 (i ) 1 1 ( 2 ,2) e

i V = V J 2 = V where we packed the transformation parameters and into according to ij = ijk k and 0i = i and where J is just the matrix dened in Peskin & Schroeder equation (3.18). So this all just says that V transforms as a four vector.

Problem 3.2
The Dirac equation for u(p) is

/ pu(p) = mu(p).

(1)

We will also need the corresponding equation for u(p). Taking the adjoint of (1), u (p)(/ p) = mu (p). On the LHS we insert a factor of 0 0 = , and we multiply both sides by 0 on the left to obtain u(p) 0 / p 0 = mu(p). (2) We have 0 0 = = (since i = i ) = 0 0 0 1 1 0 0 1 1 0 0 0 0

0 1 1 0 0 1 1 0

= .

Homer Reids Solutions to Peskin and Schroeder Problems: Chapter 3

So that was easy enough. Plugging into (2), we nd the Dirac equation for u. u(p)/ p = mu(p). Having established that preliminary result, we next observe 1 i q = [ , ] (p p ) 2 1 = p p p + p . 2 But from the anticommutation relation for the matrices we know = + 2 . Plugging this in above, we can rewrite (4) as i q = 1 2p / 2 / p + 2 p + 2 p 2 =p / + / p (p + p ) (5) (6) (3)

(4)

Finally, we just sandwich (6) between u(p) and u(p ) and use (1) and (2) to obtain u(p ) i u(p) = u(p ) p / + / p (p + p ) u(p) = u(p ) [2m (p + p )] u(p) or u(p ) u(p) = u(p ) as advertised. i q p + p u(p) + 2m 2m

Problem 3.3
(a) We have k / / k / 0 uR0 = k 0 1 uL0 Lets work out the commutator of k / / 0 and k 1 :
[k / / 0 ,k 1 ] = k0 k1 k1 k0

(7)

= k0 k1 = 2k0 k1 g

Homer Reids Solutions to Peskin and Schroeder Problems: Chapter 3

The rst term vanishes because the s anticommute, and the second term vanishes because k0 k1 = 0 : = 0. Hence we can interchange k /1 and k / 0 in (7): k / / / k / 0k 1 uL0 = k 1 0 uL0 (8)

But uL0 is the spinor for a massless fermion with momentum k0 , so its Dirac equation is just k / 0 uL0 = 0. Hence (8) vanishes. Next, 1 / puL,R (p) = / p/ pUR0,L0 = 0 2p k0 since / p/ p = p2 = 0 . (b). We have uL0 k0 = k0 + 3 = E 3 1 0 0 0 = 2E 0 0 0 1

Since k0 is the momentum of a particle moving in the negative z direction, for this to be a left-handed spinor requires that we take = 1 0 , so 1 0 uLO = 2E 0 . 0 Next, uR0 = k / 1 uL0 1 uL0 0 0 0 = 2E 0 1 = 0 1

Homer Reids Solutions to Peskin and Schroeder Problems: Chapter 3

Next, 0 0 1 uL (p) = / p p0 + p3 0 1 p1 + ip2 p0 + p3 1 = 0 p0 + p3 0 Finally, 1 0 1 / p uR (p) = p0 + p3 0 0 0 1 0 = p + p3 p0 + p3 0 p1 + ip2 (c). Using the explicit forms we found above, s(p1 , p2 ) = uR (p1 )uL (p2 ) = 1 (p0 1 +
0 p3 1 )(p2

p3 2)

3 0 p0 1 + p1

2 p1 1 ip1

3 1 2 1 2 0 3 (p0 1 + p1 )(p2 + ip2 ) + (p1 ip1 )(p2 + p2 ) 3 0 3 (p0 1 + p1 )(p2 + p2 )

2 p1 2 + ip2 3 p0 2 + p2 0 0

Separating into real and imaginary parts, =


3 1 0 3 1 0 3 2 0 3 2 (p0 2 + p2 )p1 (p1 + p1 )p2 + i (p1 + p1 )p2 (p2 + p2 )p1 3 0 3 (p0 1 + p1 )(p2 + p2 )

From this it is evident that s(p1 , p2 ) = s(p2 , p1 ). The squared magnitude is


3 1 0 3 1 3 2 0 3 2 (p0 + (p0 2 + p2 )p1 (p1 + p1 )p2 1 + p1 )p2 (p2 + p2 )p1 |s(p1 , p2 )| = 0 3 0 3 (p1 + p1 )(p2 + p2 ) 0 3 2 12 22 3 2 12 22 0 3 0 3 1 1 2 2 (p + p1 ) (p2 + p2 ) + (p0 2 + p2 ) (p1 + p1 ) 2(p2 + p2 )(p1 + p1 )(p1 p2 + p1 p2 ) = 1 0 3 0 3 (p1 + p1 )(p2 + p2 ) 0 3 0 3 p2 + p2 12 p + p1 12 22 22 1 1 2 2 (9) = 1 3 (p2 + p2 ) + p0 + p3 (p1 + p1 ) 2(p1 p2 + p1 p2 ). p0 + p 2 1 2 1 2 2 2

Homer Reids Solutions to Peskin and Schroeder Problems: Chapter 3

But since p1 is lightlike, we have


22 02 32 p12 1 + p1 = p1 p1

and similarly for p2 . Plugging these in to (9), we obtain |s(p1 , p2 )|2 =


3 3 p0 p0 2 + p2 02 1 + p1 02 32 32 1 1 2 2 ( p + p ) 2 2 3 3 (p1 + p1 ) 2(p1 p2 + p1 p2 ). p0 p0 2 + p2 1 + p1

Now simply multiplying out and simplifying we obtain


0 1 1 1 1 3 3 = 2 p0 1 p2 2 p1 p2 2 p1 p2 2 p1 p2 = 2 p1 p2 .

Problem 3.4
Part a. We suppose we have a wavefunction that we know satises the equation i (x) = im 2 (x). (10) Now suppose a friend of ours in a dierent reference frame (related to ours by a Lorentz transformation ) wanted to do some physics in his reference frame, but he didnt have values for the wavefunction in his frame, so he asked to use the values of that we have tabulated in our frame. We are happy to let him use our values, but we have to remind him to be careful. First of all, if he wants to know the value of the wavefunction at a spacetime point x in his frame, he has to ask us for the value of our wavefunction at the point 1 x in our frame. Second, once we look up the value of our wavefunction at that point and report it to him, he then has to transform the result according to P & S equation 3.37 to make sure the wavefunction has the right orientation in his reference frame. To remember all of this we can write the function (x ), which gives the value of the wavefunction in our friends frame evaluated at a point x in our friends frame, as follows: i (x ) = e 2 (i ) 1 x . Lets now see what our friend would obtain on applying the operator i to his wavefunction: i (x ) = i e 2 (i ) 1 x But is just a scalar operator and passes right through the transformation matrix: = i e 2 (i ) (1 ) | |x=1 x We now use the commutation identity e 2 (i ) = e 2 (i )
i i i i

(11)

Homer Reids Solutions to Peskin and Schroeder Problems: Chapter 3

where is the Lorentz transformation matrix corresponding to the parameters , . Using this in (11), we obtain i (x ) = i e 2 (i )
i

(1 )

(1 )

x=1 x

We can simplify the product of the matrices in the second and third square brackets with the identity = i e 2 (i ) = e
i 2 ( i ) i

(1 ) i

(1 )

x=1 x

x=1 x

But now we can just plug in from (10): = e 2 (i ) = im 2 (x ). So our friend nds his wavefunction satises the same equation ours does. Next, we write equation (10) and its complex conjugate in the form: = m 2 = m . Multiplying the rst of these by 2 /m, we solve for : = and plugging into (13) yields 1 2 = m 2 m Finally, we use the identity 2 = 2 (P & S equation (3.38)) to write 1 2 = m 2 m or ( m2 ) = 0. Part b. The action density is S= d4 x i + im T 2 2 2 . 1 2 m
2
i

im 2 1 x

(12) (13)

Homer Reids Solutions to Peskin and Schroeder Problems: Chapter 3

10

For the rst term in the action density, we take the complex conjugate and recall the rule ( ) = for Grassmann numbers: i
= 1 [i ]1 + 2 [i ]2

= i im T 2 2 2

= [i ]1 1 + [i ]2 2

(14)

The complex conjugate of the second term in the action density is =


im i1 2 + i2 1 + i 1 2 i2 1 2 im + i = 2 1 i1 2 i2 1 + i1 2 2 im T 2 =+ 2 (15) 2

so this term is its own complex conjugate. Hence we have S S = = =0 since this is the integral of a perfect dierential over all space and we assume the integrand vanishes at the surface. Next we write the action density in the form
S = 1 [i ]1 + 2 [i ]2 +

d4 x i + i d4 xi

im 2 2 1 [ 2 ]1 + 2 [ 2 ]2 1 [ ]1 2 [ ]2 2

Varying with respect to 1 and 2 we obtain

S = i [ ]1 1 S = i [ ]2 0= 2 0=

im [2 ]1 2 im [2 ]2 2

which are just the 1 and 2 components of (10). Part c. The Dirac Lagrangian density is

L = i m =i
T L T R

L R

T L

T R

L R

T T T T = iL L + iR R mL L mR R

Homer Reids Solutions to Peskin and Schroeder Problems: Chapter 3

11

In terms of 1 and 2 we have


T 2 T 2 T T L = i 1 1 + i(i 2 ) (i 2 ) m1 1 m2 2 .

We rewrite the second term:


T 2 T 2 2 i(i 2 2 ) (i 2 ) = i(2 2 )

Now, 2 2 = + if = 2 and otherwise; this is the same as - , so we obtain


= i(T 2 2 )

T = (full derivative) + i 2 2

Then the Lagrangian density is (ignoring terms that vanish on integration)


T T T T L = i 1 1 + i2 2 m1 1 m2 2 .

Part d.

Taking the divergence of the rst current, we nd J = + . (16)

The Majorana equation is i = im 2 and its adjoint is Plugging into (16), i = imT 2 . J = mT 2 + m 2 . Turning now to the second current we are given, we recall from the previous part that the terms in the Lagrangian density containing 1 and 2 are both of the same form, so we expect to get the same Majorana equation for both elds, and hence each part of the current gives a term of the form we found for the divergence of the rst current, i.e.
2 2 T 2 T 2 J = m( 1 1 1 1 ) m(2 2 2 2 ).

(17)

Problem 3.5
Part a. The Lagrangian density is L = T1 + T2 + T3 T1 = T2 = i T3 = F F.

Homer Reids Solutions to Peskin and Schroeder Problems: Chapter 3

12

We add to the elds the quantities = i =


T

2
1 2 2

2 1

= F (=

F = i = i 1 [ ]1 i

2 ) .

2 [ ]2

Since and are Grassmann, we have to be careful about taking the complex conjugate of a term in which they both appear. For example, = i = = i
1 2 1 T

2
1

2 2 1 2 2 1 2 1

0 i i 0

= 2 = =

1
T

1 2 1 2

= i = i

2 T

2 T .

Hmmm. Well, I guess we dont have to be too careful. We just have to put in a minus sign in addition to taking the normal complex conjugate of a term in which two Grassmann elds appear. To rst order in , the various terms in the Lagrangian transform are augmented as follows: T1 = i
T

2
T

T2 = i F i + iF
2 2

( 1 ) ( 3 )

( 2 )

( 4 ) ( 5 )

T3 = iF iF

i
T

( 6 ) ( 7 )

( 8 )

We proceed to argue that these terms sum to zero.

Homer Reids Solutions to Peskin and Schroeder Problems: Chapter 3

13

1. 1 and 6 These guys sum to zero because = , which follows immediately from the anticommutation relations of the matrices.

2. 2 and 4 Again we use the anticommutation relations of the matrices to obtain 4 = i 2 Then, summing in 2 , we have 2 + 4 =
T

2 i 2
T

which is the divergence of a four-vector, and hence does not aect the Lagrangian dynamics. 3. 3 and 8 These guys are i F iF = i F again the divergence of a four-vector, and hence irrelevant. 4. 5 and 7 These guys just at out cancel. Part b. The composite Lagrangian is im T 2 2 . (18) 2

L = + i +F F +mF +m F +

The Euler-Lagrange equations for F and F are L =0 F L =0 F = = F = m F = m .

Substituting in for F and F in (18), L = + i + m2 m2 m2 + = + i m2 + im T 2 2 2

im T 2 2 2

which is a Lagrangian with mass terms for both and with the same mass m.

Homer Reids Solutions to Peskin and Schroeder Problems: Chapter 3

14

Part c.

The Lagrangian density is W [] i 2 W [] T 2 + j + c.c. i 2 i j i

L = i i + i i i + Fi Fi + Fi

With the choice W = g3 /3, we have L = + i + F F + gF 2 + igT 2 + gF 2 ig 2 . The Euler-Lagrange equations for F and F are F = g2 , Plugging these in, L = + i g 2 2 2 + igT 2 ig 2 . The equation of motion for is or = 2g 2 ig 2 To get the equation we write out the dependent terms in L explicitly in terms of the components of : L = i 1 2 0 3 1 i2 1 + i2 0 + 3 1 2
+ g(1 2 2 1 ) g ( 1 2 + 2 1 )

F = g2 .

L L = ( )

= i 1 (0 3 )1 + i2 (1 i2 )1 + 2g1 2 + terms independent of 1

The the Euler-Lagrange equation is or


i(0 3 ) 1 + i(1 i2 )2 = 2g2

L L = ( 1 ) 1

This looks like the 1 component of i = 2ig2 .

Homer Reids Solutions to Peskin and Schroeder Problems: Chapter 3

15

Problem 3.6
(a). We are given the normalization for the scalar and vector elements of the set, namely scalar: vector: For the tensor elements we have tr [ , ][ , ] = ( ) ( ) = + where we employed an obvious shorthand. Since = (otherwise the whole thing vanishes) we can use the relation = to bubble the s past the s to make each term look like the rst: = 4. The trace of this is (P& S equation 5.5) tr = 16(g g g g + g g ) = 16g g = 16, +16, , both spacelike , one spacelike and one timelike

0 , i i

so the correctly normalized tensor elements of the {} set are 1 + [ 0 , 1 ], 4 tensor: 1 [ 1 , 3 ], 4 For the pseudovectors we have 1 + [ 0 , 2 ], 4 1 2 3 [ , ], 4
2

1 + [ 0 , 3 ], 4 1 [ 1 , 2 ]. 4

tr 5 5 = tr 2 5 = tr g = 4, 4,

=0 = 1, 2, 3

so the correctly normalized pseudovector elements are pseudovector : 0 5 , i i 5

Finally, for the pseudoscalar element we already have tr 5 5 = 4, so the correctly normalized pseudoscalar element of the {} set is pseudoscalar : 5.

Das könnte Ihnen auch gefallen